Tải bản đầy đủ (.pdf) (38 trang)

định lý sylow và bài tập vận dụng

Bạn đang xem bản rút gọn của tài liệu. Xem và tải ngay bản đầy đủ của tài liệu tại đây (326.24 KB, 38 trang )

BÀI TẬP LÝ THUYẾT NHÓM
Đặng Tuấn Hiệp
Cao học K17
Đại số và lý thuyết số
Tháng 10 năm 2007
1
1
PGS-TS Mỵ Vinh Quang
Mục lục
1 BÀI TẬP ĐỊNH LÝ SYLOW 2
4 BÀI TẬP CHƯƠNG 4 10
4.1 Nhóm xoắn và nhóm chia được . . . . . . . . . . . . . . . . . . . . . . . . 10
4.2 Tổng trực tiếp các nhóm cyclic và tựa cyclic . . . . . . . . . . . . . . . . 20
1
Chương 1
BÀI TẬP ĐỊNH LÝ SYLOW
Đề bài
1. Cho G là nhóm đơn và có nhóm con H có chỉ số n trong G. Chứng minh
rằng G là nhóm hữu hạn và |G||n!
2. Chứng minh rằng nhóm cấp p
n
(n>1) và cấp pq (p = q) đều không là nhóm
đơn.
3. Chứng minh rằng nhóm cấp p
2
q không là nhóm đơn.
4. Chứng minh rằng nhóm cấp pqr không là nhóm đơn.
(p, q, r là các số nguyên tố đôi một khác nhau)
5. Chứng minh rằng nhóm cấp 24, 36 không là nhóm đơn.
6. Chứng minh rằng nhóm cấp 56 không là nhóm đơn.
7. Chứng minh rằng nhóm cấp 72, 80, 96, 108, 150, 154, 160 không là nhóm đơn.


8. Cho G là p−nhóm hữu hạn, H ≤ G, H = G. Chứng minh rằng H = N
G
(H).
9. Chứng minh rằng nhóm cấp 132 không là nhóm đơn.
10. Chứng minh rằng nhóm cấp 144 không là nhóm đơn.
Bài giải chi tiết
Bài 1.0.1. Cho G là nhóm đơn và có nhóm con H có chỉ số n>1 trong G. Chứng
minh rằng G là nhóm hữu hạn và |G||n!
2
Đặng Tuấn Hiệp - Bài tập lý thuyết nhóm 3
Giải. Đặt X = {xH : x ∈ G}. Khi đó |X| =(G : H)=n. Với mọi g ∈ G, xét
tương ứng ¯g : X −→ X; xH −→ gxH . Ta có x
1
H = x
2
H ⇔ (x
1
)
−1
x
2
∈ H ⇔
((x
1
)
−1
g
−1
)(gx
2

) ∈ H ⇔ ((gx
1
)
−1
)(gx
2
) ∈ H ⇔ gx
1
H = gx
2
H. Do đó ¯g là ánh xạ
và đơn ánh, mà X là hữu hạn nên ¯g là song ánh, suy ra ¯g ∈ S
X
. Xét ánh xạ
f : G −→ S
X
; g −→ ¯g
∀g
1
,g
2
∈ G thì g
1
g
2
=¯g
1
¯g
2
. Thật vậy, ∀xH ∈ X ta có

g
1
g
2
(xH)=g
1
g
2
xH =¯g
1
¯g
2
(xH)
Do đó f là đồng cấu nhóm.
Ta có ker f  G.VìG là nhóm đơn nên ker f = G hoặc ker f =0. Nếu ker f = G thì
¯g =1
X
; ∀g ∈ G, suy ra gH = H; ∀g ∈ G ⇒ g ∈ H; ∀g ∈ G ⇒ G = H (mâu thuẫn).
Do đó ker f =0hay f là đơn cấu. Suy ra G

=
f(G) ≤ S
X
hay |G|||S
X
| = n!
Bài 1.0.2. Chứng minh rằng nhóm cấp p
n
(n>1) và cấp pq (p = q) đều không là
nhóm đơn.

Giải.
• Lấy G là nhóm có cấp p
n
(n>1).
Cách 1: Giả sử G là nhóm đơn. Theo đònh lý Sylow, tồn tại p−nhóm con H
có cấp p
n−1
, khi đó (G : H)=p>1. Theo bài 1, |G||p! hay p
n
| p!. Điều này
không thể xảy ra được với n>1.
Cách 2:
Theo công thức lớp ta có
p
n
= |G| = |Z(G)| +
n

i=1
(G : C(x
i
))
Do |G|
.
.
. p và (G : C(x
i
))
.
.

. p nên |Z(G)|
.
.
. p hay |Z(G)| > 1. Vậy p−nhóm G có
tâm không tầm thường và đó là nhóm con chuẩn tắc không tầm thường của
G. Suy ra G không là nhóm đơn.
• Lấy G là nhóm có cấp pq, với p và q là các số nguyên tố khác nhau, giả sử
p<q. Gọi n
q
là số các q−nhóm con Sylow của G. Theo đònh lý Sylow, ta có

n
q
|p
n
q
≡ 1 (mod q)
Suy ra n
q
=1. Gọi H là q−nhóm con Sylow duy nhất của G. Khi đó xHx
−1
=
H; ∀x ∈ G ⇒ H  G. Do đó G không là nhóm đơn.
Đặng Tuấn Hiệp - Bài tập lý thuyết nhóm 4
Bài 1.0.3. Chứng minh rằng nhóm cấp p
2
q không là nhóm đơn.
Giải. Lấy G là nhóm có cấp p
2
q, với p và q là các số nguyên tố khác nhau. Giả sử G

là nhóm đơn. Gọi n
p
,n
q
lần lượt là số các p−nhóm con Sylow, q−nhóm con Sylow
của G. Khi đó n
p
> 1,n
q
> 1. Theo đònh lý Sylow, ta có

n
p
| q
n
p
≡ 1 (mod p)
⇒ n
p
= q và q>p


n
q
| p
2
n
q
≡ 1 (mod q)
⇒ n

q
= p
2
Nếu Q
1
,Q
2
đều có cấp q và Q
1
= Q
2
thì Q
1
∩ Q
2
= {e}. Do đó số phần tử của G có
cấp q là n
q
(q − 1) = p
2
(q − 1) = p
2
q − p
2
. Suy ra số phần tử của G không có cấp q
là p
2
. Do đó n
p
=1(mâu thuẫn với n

p
> 1). Vậy G không là nhóm đơn.
Bài 1.0.4. Chứng minh rằng nhóm cấp pqr không là nhóm đơn.
Giải. Lấy G là nhóm có cấp pqr, với p, q, r là các số nguyên tố đôi một khác nhau.
Giả sử G là nhóm đơn và p<q<r. Gọi n
p
,n
q
,n
r
lần lượt là số các p−nhóm con
Sylow, q−nhóm con Sylow, r−nhóm con Sylow của G. Khi đó n
p
> 1,n
q
> 1,n
r
> 1.
Theo đònh lý Sylow, ta có

n
r
| pq
n
r
≡ 1 (mod r)
⇒ n
r
= pq


n
q
| pr
n
q
≡ 1 (mod q)
⇒ n
q
= pr hoặc n
q
= r ⇒ n
q
≥ r

n
p
| qr
n
p
≡ 1 (mod p)
⇒ n
p
= qr hoặc n
p
= q hoặc n
p
= r ⇒ n
p
≥ q
Suy ra số phần tử cấp r là pq(r − 1).

Số phần tử cấp q ít nhất là r(q − 1).
Số phần tử cấp p ít nhất là q(p − 1).
Do đó số phần tử của G ít nhất phải là pq(r − 1) + r(q − 1) + q(p − 1) = pqr +(r −
1)(q − 1) − 1 >pqr(mâu thuẫn vì |G| = pqr)
Bài 1.0.5. Chứng minh rằng nhóm cấp 24, 36 không là nhóm đơn.
Giải.
Đặng Tuấn Hiệp - Bài tập lý thuyết nhóm 5
• Lấy G là nhóm cấp 24 = 2
3
.3. Giả sử G là nhóm đơn. Gọi n
2
,n
3
lần lượt là
số các 2−nhóm con Sylow, 3−nhóm con Sylow của G. Khi đó n
2
> 1,n
3
> 1.
Theo đònh lý Sylow, ta có

n
2
| 3
n
2
≡ 1 (mod 2)
⇒ n
2
=3

Gọi H là một 2−nhóm con Sylow của G, ta có
(G : N
G
(H)) = n
2
=3
Theo bài bập 1, |G||3! (vô lý)
• Lấy G là nhóm cấp 36 = 2
2
.3
2
. Giả sử G là nhóm đơn. Gọi n
2
,n
3
lần lượt là
số các 2−nhóm con Sylow, 3−nhóm con Sylow của G. Khi đó n
2
> 1,n
3
> 1.
Theo đònh lý Sylow, ta có

n
3
| 4
n
3
≡ 1 (mod 3)
⇒ n

3
=4
Gọi H là một 3−nhóm con Sylow của G, ta có
(G : N
G
(H)) = n
3
=4
Theo bài bập 1, |G||4! (vô lý)
Chú ý. N
G
(H) là chuẩn hóa tử của H trong G, tức là nhóm con lớn nhất trong G
nhận H làm nhóm con chuẩn tắc. Liên quan đến nhóm chuẩn hóa tử ta có các
mệnh đề sau.
Mệnh đề 1. Nếu G là một nhóm hữu hạn với H ≤ G thì số các nhóm con của G liên
hợp với H bằng chỉ số của N
G
(H) trong G.
Bài 1.0.6. Chứng minh rằng nhóm cấp 56 không là nhóm đơn.
Giải. Lấy G là nhóm cấp 56=2
3
.7. Giả sử G là nhóm đơn. Gọi n
2
,n
7
lần lượt là
số các 2−nhóm con Sylow, 7−nhóm con Sylow của G. Khi đó n
2
> 1,n
7

> 1. Theo
đònh lý Sylow, ta có

n
7
| 8
n
7
≡ 1 (mod 7)
⇒ n
7
=8
Số phần tử cấp 7 là 8(7 − 1) = 48.
Số phần tử cấp khác 7 là 56 − 48=8.
Ta lại có, mỗi 2−nhóm con Sylow có 2
3
=8phần tử cấp khác 7. Do đó n
2
=1(mâu
thuẫn với n
2
> 1)
Đặng Tuấn Hiệp - Bài tập lý thuyết nhóm 6
Bài 1.0.7. Chứng minh rằng nhóm cấp 72, 80, 96, 108, 150, 154, 160 không là nhóm đơn.
Giải.
• Lấy G là nhóm cấp 72 = 2
3
.3
2
. Giả sử G là nhóm đơn. Gọi n

3
là số các
3−nhóm con Sylow của G. Khi đó n
3
> 1. Theo đònh lý Sylow, ta có

n
3
| 8
n
3
≡ 1 (mod 3)
⇒ n
3
=4
Gọi H là một 3−nhóm con Sylow của G, ta có
(G : N
G
(H)) = n
3
=4
Theo bài bập 1, |G||4! (vô lý)
• Lấy G là nhóm cấp 80 = 2
4
.5. Giả sử G là nhóm đơn. Gọi n
2
,n
5
lần lượt là
số các 2−nhóm con Sylow, 5−nhóm con Sylow của G. Khi đó n

2
> 1,n
5
> 1.
Theo đònh lý Sylow, ta có

n
5
| 16
n
5
≡ 1 (mod 5)
⇒ n
5
=16
Số phần tử cấp 5 là 16(5 − 1) = 64.
Số phần tử cấp khác 5 là 80 − 64 = 16.
Ta lại có, mỗi 2−nhóm con Sylow có 2
4
=16phần tử cấp khác 5. Do đó
n
2
=1(mâu thuẫn với n
2
> 1)
• Lấy G là nhóm cấp 96 = 2
5
.3. Giả sử G là nhóm đơn. Gọi n
2
là số các 2−nhóm

con Sylow của G. Khi đó n
2
> 1. Theo đònh lý Sylow, ta có

n
2
| 3
n
2
≡ 1 (mod 2)
⇒ n
2
=3
Gọi H là một 2−nhóm con Sylow của G, ta có
(G : N
G
(H)) = n
2
=3
Theo bài bập 1, |G||3! (vô lý)
• Lấy G là nhóm cấp 108 = 2
2
.3
3
. Giả sử G là nhóm đơn. Gọi n
3
là số các
3−nhóm con Sylow của G. Khi đó n
3
> 1. Theo đònh lý Sylow, ta có


n
3
| 4
n
3
≡ 1 (mod 3)
⇒ n
3
=4
Đặng Tuấn Hiệp - Bài tập lý thuyết nhóm 7
Gọi H là một 3−nhóm con Sylow của G, ta có
(G : N
G
(H)) = n
3
=4
Theo bài bập 1, |G||4! (vô lý)
• Lấy G là nhóm cấp 150 = 2.3.5
2
. Giả sử G là nhóm đơn. Gọi n
5
là số các
5−nhóm con Sylow của G. Khi đó n
5
> 1. Theo đònh lý Sylow, ta có

n
5
| 6

n
5
≡ 1 (mod 5)
⇒ n
5
=6
Gọi H là một 5−nhóm con Sylow của G, ta có
(G : N
G
(H)) = n
5
=6
Theo bài bập 1, |G||6! (vô lý)
• Lấy G là nhóm cấp 154 = 2.7.11. Có thể p dụng bài tập 4 hoặc có thể làm
trực tiếp như sau.
Giả sử G là nhóm đơn. Gọi n
11
là số các 11−nhóm con Sylow của G. Khi đó
n
11
> 1. Theo đònh lý Sylow, ta có

n
11
| 14
n
11
≡ 1 (mod 14)
⇒ n
11

=1(mâu thuẫn với n
11
> 1)
• Lấy G là nhóm cấp 160 = 2
5
.5. Giả sử G là nhóm đơn. Gọi n
2
là số các
2−nhóm con Sylow của G. Khi đó n
2
> 1. Theo đònh lý Sylow, ta có

n
2
| 5
n
2
≡ 1 (mod 2)
⇒ n
2
=5
Gọi H là một 2−nhóm con Sylow của G, ta có
(G : N
G
(H)) = n
2
=5
Theo bài bập 1, |G||5! (vô lý)
Bài 1.0.8. Cho G là p−nhóm hữu hạn, H ≤ G, H = G. Chứng minh rằng H = N
G

(H).
Giải.
Bài 1.0.9. Chứng minh rằng nhóm cấp 132 không là nhóm đơn.
Đặng Tuấn Hiệp - Bài tập lý thuyết nhóm 8
Giải. Lấy G là nhóm cấp 132 = 2
2
.3.11. Giả sử G là nhóm đơn. Gọi n
2
,n
3
,n
11
lần
lượt là số các 2−nhóm con Sylow, 3−nhóm con Sylow, 11−nhóm con Sylow của G.
Khi đó n
2
> 1,n
3
> 1,n
11
> 1. Theo đònh lý Sylow, ta có

n
11
| 12
n
11
≡ 1 (mod 11)
⇒ n
11

=12

n
3
| 44
n
3
≡ 1 (mod 3)
⇒ n
3
=22hoặc n
3
=4
Số phần tử cấp 11 là 12(11 − 1) = 120.
Số phần tử cấp khác 11 là 132 − 120 = 12
Nếu n
3
=22thì số phần tử cấp 3 là 22(3 − 1) = 44 > 12 (mâu thuẫn). Do đó
n
3
=4, số phần tử cấp 3 là 4(3 − 1) = 8. Suy ra, số phần tử cấp khác 11 và khác 3
là 12 − 8=4. Ta lại có, mỗi 2−nhóm con Sylow có 2
2
=4phần tử cấp khác 3 và
khác 11. Do đó n
2
=1(mâu thuẫn với n
2
> 1)
Cách 2. Sau khi đã lý luận để có được n

3
=4. Gọi H là một 3−nhóm con Sylow
của G, ta có
(G : N
G
(H)) = n
3
=4
Theo bài bập 1, |G||4! (vô lý)
Bài 1.0.10. Chứng minh rằng nhóm cấp 144 không là nhóm đơn.
Giải. Lấy G là nhóm cấp 144 = 2
4
.3
2
. Giả sử G là nhóm đơn. Gọi n
2
,n
3
lần lượt là
số các 2−nhóm con Sylow, 3−nhóm con Sylow của G. Khi đó n
2
> 1,n
3
> 1. Theo
đònh lý Sylow, ta có

n
3
| 16
n

3
≡ 1 (mod 3)
⇒ n
3
=16hoặc n
3
=4
Nếu n
3
=4thì gọi H là một 3−nhóm con Sylow của G, ta có
(G : N
G
(H)) = n
3
=4
Theo bài bập 1, |G||4! (vô lý)
Nếu n
3
=16thì xét các trường hợp sau:
1. Hai 3−nhóm con Sylow bất kỳ có giao chỉ gồm đơn vò. Khi đó số phần tử cấp
3 hoặc cấp 9 là 16(9 − 1) = 128. Số phần tử còn lại là 144 − 128 = 16. Ta lại
có, mỗi 2−nhóm con Sylow có 2
4
=16phần tử. Do đó n
2
=1(mâu thuẫn với
n
2
> 1)
2. Tồn tại hai 3−nhóm con Sylow P,Q sao cho P ∩ Q = T và |T | =3. Trước

tiên, ta chứng minh bổ đề sau:
Đặng Tuấn Hiệp - Bài tập lý thuyết nhóm 9
Bổ đề 1. Nếu G là nhóm cấp p
2
với p là số nguyên tố thì G là nhóm Abel.
Chứng minh. Gọi Z(G) là tâm của nhóm G. Ta chứng minh Z(G)=G. Giả
sử ngược lại, Z(G) = G. Vì trong một p−nhóm, nhóm con Z(G) là nhóm con
chuẩn tắc không tầm thường, nên |Z(G)| = p. Xét nhóm thương G/Z(G) có p
phần tử. Giả sử Z(G)= a
p
,G/Z(G)=b
p
. Suy ra G = a, b. Ta có ab = ba
(vì a ∈ Z(G)), do đó G là nhóm Abel, hay Z(G)=G (trái với giả thiết phản
chứng). Vậy G là nhóm Abel.
Trở lại bài toán, ta có |P | = |Q| =3
2
nên P, Q là các nhóm Abel. Suy ra
T  P, T  Q hay P, Q ≤ N
G
(T ) ⇒|N
G
(T )|≥18 (vì nếu |N
G
(T )| =9thì P =
N
G
(T )=Q (mâu thuẫn với P = Q). Ta lại có N
G
(T ) ≤ G ⇒|N

G
(T )||144.
Do đó |N
G
(T )|∈{18, 36, 72, 144}.
•|N
G
(T )| =18=2.3
2
⇒ N
G
(T ) chỉ có một 3−nhóm con Sylow (mâu thuẫn
với P = Q)
•|N
G
(T )| =36⇒ (G : N
G
(T )) = 4 ⇒|G||4! (vô lý)
•|N
G
(T )| =72⇒ (G : N
G
(T )) = 2 ⇒|G||2! (vô lý)
•|N
G
(T )| = 144 ⇒ N
G
(T )=G ⇒ T  G (mâu thuẫn với G là nhóm đơn)
Chương 4
BÀI TẬP CHƯƠNG 4

Chúng ta luôn có giả thiết các nhóm được xét tới đều là nhóm Abel.
4.1 Nhóm xoắn và nhóm chia được
Đề bài
1. Chứng minh rằng nhóm kiểu p

có duy nhất một nhóm con có cấp p
i
và nhóm
này là nhóm cyclic. Chứng minh rằng mọi nhóm con thật sự của p

−nhóm
đều là nhóm hữu hạn.
2. Cho G là nhóm Abel vô hạn thỏa điều kiện mọi nhóm con thật sự của G đều
hữu hạn. Chứng minh rằng G là p

−nhóm.
3. Cho G là nhóm Abel vô hạn thỏa điều kiện mọi nhóm thương thật sự của G
đều hữu hạn. Chứng minh rằng G là nhóm cyclic vô hạn.
4. Chứng minh rằng nhóm G là nhóm chia được khi và chỉ khi G

=
H ≤ K ⇒ H
là hạng tử trực tiếp của K.
5. Mô tả cấu trúc của các nhóm R, R

, C, C

.
6. Cho G là nhóm Abel kiểu p−nhóm thỏa G/G[p] là nhóm chia được. Chứng
minh rằng G là tổng trực tiếp của một nhóm chia được và một nhóm Abel sơ

cấp kiểu p−nhóm.
7. Cho G, H là các p−nhóm chia được. Chứng minh rằng G

=
H ⇔ G[p]

=
H[p].
8. Chứng minh rằng G chia được khi và chỉ khi G không có nhóm con tối đại.
10
Đặng Tuấn Hiệp - Bài tập lý thuyết nhóm 11
9. Chứng minh rằng G chia được khi và chỉ khi G không có ảnh đồng cấu hữu
hạn khác 0.
10. Lấy ví dụ về hai nhóm con chia được nhưng có giao không chia được.
Bài giải chi tiết
Bài 4.1.1. Chứng minh rằng nhóm kiểu p

có duy nhất một nhóm con có cấp p
i
, với
mỗi i =1, 2, và nhóm này là nhóm cyclic. Chứng minh rằng mọi nhóm con thật sự
của p

−nhóm đều là nhóm hữu hạn.
Nhắc lại rằng, nhóm tựa cyclic kiểu p là nhóm sinh bởi a
1
,a
2
, ,a
n

, với quan
hệ:
pa
1
=0,pa
i+1
= a
i
,a
i
+ a
j
= a
j
+ a
i
Tức là nhóm P = a
1
,a
2
, ,a
n
,  trong đó a
i
có cấp p
i
và a
i
 là nhóm cyclic
cấp p

i
. Hơn nữa, a
1
= p
i−1
a
i
(i ≥ 2) và a
i
= p
j−i
a
j
(1 ≤ i ≤ j), điều này có nghóa
là hai phần tử bất kỳ trong tập các phần tử sinh luôn phụ thuộc.
Nhóm tựa cyclic kiểu p có một minh họa khá đẹp trong Q/Z như sau:
P = 
1
p
i
+ Z | i =1, 2, 
Giải. Gọi G là nhóm tựa cyclic kiểu p sinh bởi a
1
,a
2
, ,a
n
, Lấy H là một nhóm
con thật sự của G, ta sẽ chứng minh H = a
n

, với n là số nguyên dương nào đó. Do
H = G nên tồn tại số nguyên dương i sao cho a
i
/∈ H, suy ra a
k
/∈ H, ∀k ≥ i. Gọi n
là số nguyên dương lớn nhất để a
n
∈ H (1 ≤ n<i). Ta sẽ chứng minh H = a
n

• Vì a
n
∈ H nên  a
n
⊂H
• Lấy h ∈ H, khi đó h =

k
i=1
m
i
a
i
, giả sử m
k
=0⇒ h = la
k
(Cụ thể l = m
1

p
k−1
+ m
2
p
k−2
+ ···+ m
k−1
p + m
k
)
Ta có thể giả sử (l,p)=1, vì nếu ngược lại, h = l
1
a
k−1
và ta lại giả sử (l
1
,p)=1,
vì nếu ngược lại, h = l
2
a
k−2
, và cứ thế, ta phải tìm được số nguyên dương t
sao cho h = l
t
a
t
với (l
t
,p)=1.Do(l, p)=1và la

k
∈ H nên a
k
∈ H. Thật vậy,
ta sẽ chứng minh a
k
∈ H dựa theo một kỹ thuật thường dùng trong p−nhóm.
Do (l,p)=1⇒ (l, p
k
)=1⇒ tồn tại u, v ∈ Z : ul + vp
k
=1⇒ a
k
=1.a
k
=
(ul + vp
k
)a
k
= ula
k
+ vp
k
a
k
= u(la
k
)=uh ∈ H. Suy ra k ≤ n (vì tính lớn nhất
của n). Khi đó, a

k
= p
n−k
a
n
∈a
n
⇒h = la
k
∈a
n
. Do đó H ⊂a
n
.
Vậy H = a
n
 và ta có điều phải chứng minh.
Đặng Tuấn Hiệp - Bài tập lý thuyết nhóm 12
Tất cả các nhóm con của p−nhóm tựa cyclic là :
0 ⊂a
1
⊂a
2
⊂···⊂a
n
⊂···⊂a
1
,a
2
, ,a

n
, 
Hiển nhiên, a
i
 là nhóm cyclic cấp p
i
.
Bài 4.1.2. Cho G là nhóm Abel vô hạn thỏa điều kiện mọi nhóm con thật sự của G
đều hữu hạn. Chứng minh rằng G là nhóm tựa cyclic kiểu p.
Giải.
• G là nhóm xoắn vì nếu có a ∈ G, a là phần tử không xoắn thì a
2
 vô hạn và
a
2
⊂a≤G trái giả thiết.
• Theo đònh lý về sự phân tích nguyên sơ, ta có
G =

p
G
p
Ta sẽ chứng minh G = G
p
nào đó và trong tổng trên, tất cả các hạng tử đều
bằng 0, trừ một hạng tử G
p
=0.
Vì mọi nhóm con thật sự của G đều hữu hạn nên chỉ có hữu hạn các G
p

=0.
Thật vậy, nếu có vô hạn G
p
=0. Xét G
p
0
=0, khi đó H =

p=p
0
G
p
là nhóm
con thật sự và vô hạn của G, trái giả thiết.
• Tiếp theo, ta chứng minh chỉ có một nhóm G
p
=0.
Thật vậy, vì G =

hữu hạn
G
p
, và |G| vô hạn nên tồn tại số nguyên tố p sao
cho G
p
vô hạn. Khi đó, ta có G
p
vô hạn, G
p
≤ G, nên suy ra G

p
= G. Vậy G
là p−nhóm.
• Tiếp theo, ta chứng minh G là chia được, điều này tương đương với pG = G.
Nếu pG = G thì pG hữu hạn, G/pG vô hạn nên là không gian vectơ vô hạn
chiều trên Z
p
với phép nhân ngoài:
Z
p
× G/pG −→ G/pG
(¯n, ¯g) → ng
Gọi {α
i
}
i∈I
là cơ sở của G/pG trên Z
p
, I vô hạn.
Ta kiểm tra G = α
i
,pG
i∈I
.
Hiển nhiên, α
i
,pG
i∈I
⊂ G.
Lấy g ∈ G ⇒ ¯g ∈ G/pG, tồn tại

m
1
, ,m
k
sao cho
¯g =
k

i=1
m
i
α
i
=
k

i=1
m
i
α
i
Đặng Tuấn Hiệp - Bài tập lý thuyết nhóm 13
Suy ra
g −
k

i=1
m
i
α

i
∈ pG
g −
k

i=1
m
i
α
i
= pg
1
(g
1
∈ pG)
g =
k

i=1
m
i
α
i
+ pg
1
∈α
i
,pG
i∈I
Xét nhóm con

H = α
i
,pG
i∈I\{i
0
}
Ta sẽ chứng minh H là nhóm con thật sự của G, điều này có nghóa là α
i
0
/∈ H.
Thật vậy, giả sử α
i
0
∈ H, khi đó ta có
α
i
0
=

i=i
0
m
i
α
i
+ pg
Suy ra
α
i
0

=

i=i
0
m
i
α
i
Điều này mâu thuẫn với tính độc lập của {α
i
}
i∈I
.
Như vậy, ta đã chứng minh được H là nhóm con thật sự của G và H là vô
hạn, điều này trái với giả thiết.
Vậy ta phải có pG = G.
• Do G là p−nhóm nên tồn tại trong G phần tử a
1
có cấp p.
Vì pG = G nên tồn tại a
2
sao cho pa
2
= a
1
. Tiếp tục, lại tồn tại a
3
sao cho
pa
3

= a
2
. Một cách tổng quát, tồn tại a
n
sao cho pa
n
= a
n−1
với mọi n ≥ 2.
Xét nhóm con H = a
1
,a
2
, ,a
n
, ≤G.
Dễ thấy, H có vô hạn phần tử. Do đó, theo giả thiết, ta phải có H = G.
Vậy G là nhóm tựa cyclic kiểu p.
Chú ý. G là nhóm tựa cyclic kiểu p khi và chỉ khi G là p−nhóm, có vô hạn phần
tử, chia được (pG = G) và mọi nhóm con thật sự của G đều hữu hạn.
Bài 4.1.3. Cho G là nhóm Abel vô hạn thỏa điều kiện mọi nhóm thương thật sự của G
đều hữu hạn. Chứng minh rằng G là nhóm cyclic vô hạn. Chiều đảo có đúng không?
Giải.
Đặng Tuấn Hiệp - Bài tập lý thuyết nhóm 14
• G là nhóm không xoắn vì nếu có a ∈ G, a =0,a xoắn thì a hữu hạn và khi
đó G/a vô hạn, trái giả thiết.
• Lấy a, b ∈ G, ta sẽ chứng minh a, b là nhóm cyclic.
Thật vậy, ta có
¯
b ∈ G/a hữu hạn nên tồn tại số nguyên dương k sao cho

k
¯
b =
¯
0 ⇒ k
¯
b ∈a⇒kb = la.
Gọi d =(k, l) ⇒ k = dk
1
,l = dl
1
với (k
1
,l
1
)=1.
Ta có kb−la =0⇒ d(k
1
b−l
1
a)=0.DoG là nhóm không xoắn nên k
1
b = l
1
a.
Vậy ta có thể giả sử (k, l)=1. Khi đó, tồn tại u, v ∈ Z sao cho uk + vl =1.
Suy ra
a =(uk + vl)a = uka + v(la)=uka + vkb = k(ua + vb)
b =(uk + vl)b = u(kb)+vlb = ula + vlb = l(ua + vb)
Đặt c = ua + vb, khi đó a = kc,b = lc. Ta sẽ chứng minh a, b = c.

{ Ta có a = kc ∈c,b = lc ∈c do đó a, b⊂c.
{ Ta có c = ua + vb ∈a, b⇒c⊂a, b.
• Tiếp theo, ta sẽ chứng minh G là nhóm hữu hạn sinh.
Lấy 0 = a
1
∈ G, khi đó, theo giả thiết ta có G/a
1
 hữu hạn nên
G/a
1
 = a
2
, ,a
n

Dễ dàng, suy ra được
G = a
1
,a
2
, ,a
n

Như vậy, ta đã chứng minh được G = a
1
,a
2
, ,a
n
 và a, b = G = c. Do đó,

ta phải có G = d là nhóm cyclic vô hạn.
Chú ý. Ngược lại, giả sử G = x với x là phần tử không xoắn. Lấy H là nhóm con
thật sự của G. Ta sẽ chứng minh H = sx với số nguyên dương s nào đó.
• Do H =0nên có sx ∈ H, sx =0. Gọi s là số nguyên dương nhỏ nhất có tính
chất trên. Do sx ∈ H nên sx⊂H.
• Ngược lại, lấy h ∈ H, khi đó h = tx với t là số nguyên dương nào đó. Theo
thuật toán Euclide, ta có t = qs + r với 0 ≤ r<s. Suy ra rx = tx − q(sx)=
h − q(sx) ∈ H. Do tính nhỏ nhất của s nên ta phải có r =0, khi đó t = qs.
Suy ra tx = q(sx) ∈sx. Do đó H ⊂sx
Đặng Tuấn Hiệp - Bài tập lý thuyết nhóm 15
Vậy H = sx là nhóm cyclic vô hạn (do x không xoắn nên sx cũng không xoắn).
Khi đó, dễ thấy G/H chỉ có hữu hạn phần tử
G/H = {0+sx,x+ sx, 2x + sx, ,(s − 1)x + sx}
Như vậy, ta có thể khẳng đònh rằng: G là nhóm cyclic vô hạn khi và chỉ khi mọi
nhóm con thật sự của G đều có chỉ số hữu hạn.
Bài 4.1.4. Chứng minh rằng nhóm G là nhóm chia được khi và chỉ khi G

=
H ≤ K ⇒
H là hạng tử trực tiếp của K.
Giải.
1. Giả sử G là nhóm chia được. Nếu có các nhóm H, K sao cho G

=
H ≤ K thì
do G chia được nên H cũng chia được và do đó H là hạng tử trực tiếp của K.
2. Giả sử ta có mệnh đề: Nếu G

=
H ≤ K thì H là hạng tử trực tiếp của K.

Theo tính chất của nhóm Abel chia được, ta có mỗi nhóm Abel G có thể được
xem như là nhóm con của một nhóm Abel chia được nào đó. Điều này có
nghóa là, tồn tại nhóm Abel K chia được và nhóm H ≤ K sao cho G

=
H ≤ K.
Theo giả thiết, khi đó H là hạng tử trực tiếp của K. Suy ra tồn tại nhóm L
sao cho K = H ⊕ L. Khi đó, với mọi số nguyên dương n ta có
H = K ∩ H = nK ∩ H =(nH + nL) ∩ H =(nH ∩ H)+(nL ∩ H)=nH
Do đó H là nhóm chia được, suy ra G là nhóm chia được.
Chú ý. Nhóm con của nhóm chia được chưa chắc là nhóm chia được. Tuy nhiên,
hạng tử trực tiếp của nhóm chia được là nhóm chia được.
Bài 4.1.5. Mô tả cấu trúc của các nhóm R, R

, C, C

.
Giải.
Bài 4.1.6. Cho G là nhóm Abel kiểu p−nhóm thỏa G/G[p] là nhóm chia được. Chứng
minh rằng G là tổng trực tiếp của một nhóm chia được và một nhóm Abel sơ cấp kiểu
p−nhóm.
Nhóm Abel sơ cấp là nhóm Abel xoắn sao cho ∀a ∈ G : |a|
.
.
. p
2
với p là số
nguyên tố bất kỳ. Nghóa là |a| = p
1
p

2
p
n
, với các p
i
là các số nguyên tố đôi một
khác nhau.
Đặc biệt, nếu G là p−nhóm thì
G là nhóm Abel sơ cấp ⇔|a|≤p ⇔ pa =0
hay
p − nhóm Abel G là sơ cấp ⇔ pG =0
Đặng Tuấn Hiệp - Bài tập lý thuyết nhóm 16
Giải. Xét ánh xạ
f : G −→ pG
g −→ pg
• Dễ thấy, f là toàn cấu.
∀a, b ∈ G : f(a + b)=p(a + b)=pa + pb = f(a)+f(b)
Lấy a ∈ pG : a = pg với g ∈ G. Khi đó f(g)=a.
• Kerf = {g ∈ G : f(g)=0} = {g ∈ G : pg =0} = G[p].
Theo đònh lý Noetherian, ta suy ra
pG

=
G/G[p]
Theo giả thiết, G/G[p] chia được nên pG chia được và do đó pG là hạng tử trực tiếp
của G. Suy ra
G = pG ⊕ E
Ta sẽ chứng minh E là p−nhóm Abel sơ cấp. Thật vậy, lấy a ∈ E : pa ∈ E ∩pG =0.
Do đó, pE =0.
Bài 4.1.7. Cho G, H là các p−nhóm chia được. Chứng minh rằng

G

=
H ⇔ G[p]

=
H[p]
Giải.
1. Giả sử G

=
H, khi đó tồn tại đẳng cấu f : G → H. Do đơn cấu bảo toàn cấp
của các phần tử nên f|
G[p]
: G[p] → H[p].Vìf đơn cấu nên f|
G[p]
đơn cấu.
Ta sẽ chứng minh f|
G[p]
toàn cấu. Thật vậy, lấy h ∈ H[p] ≤ H, khi đó sẽ tồn
tại g ∈ G sao cho h = f(g). Ta có
f(0) = 0 = ph = pf(g)=f(pg)
do f đơn cấu nên pg =0, suy ra g ∈ G[p].
Vậy G[p]

=
H[p].
2. Giả sử G[p]

=

H[p], khi đó tồn tại đẳng cấu α : G[p] → H[p].
Cách 1: Xét hai đồng cấu nhúng i : H[p] → H và j : G[p] → G, khi đó i, j
đều là các đơn cấu. Xét biểu đồ sau đây
0 −−−→ G[p]
α
−−−→ H[p] −−−→ 0



j



i
GH
Đặng Tuấn Hiệp - Bài tập lý thuyết nhóm 17
Vì H là nhóm chia được nên H có tính nội xạ. Do đó, tồn tại đồng cấu
f : G → H sao cho fj = iα, tức là ta có biểu đồ giao hoán
0 −−−→ G[p]
α
−−−→ H[p] −−−→ 0



j



i
G

f
−−−→ H
Ta sẽ chứng minh f là đẳng cấu.
• f là đơn cấu.
Giả sử Kerf =0, suy ra tồn tại x =0sao cho f(x)=0.Dox =0nên
|x| = p
n
với n ≥ 1. Đặt y = p
n−1
x, khi đó y ∈ G[p] và y =0. Ta có
0=p
n−1
f(x)=f(p
n−1
x)=f(y)=fj(y)=iα(y)=α(y)
Do α là đẳng cấu nên ta phải có y =0(mâu thuẫn với y =0)
• f là toàn cấu.
Ta có Imf = f(G) ≤ H,doG là nhóm chia được nên f(G) là nhóm
con chia được của H. Suy ra tồn tại K ≤ H sao cho H = f(G) ⊕ K ⇒
H[p]=f(G)[p] ⊕ K[p]. Giả sử f(G) = H ⇒ K =0⇒ K[p] =0. Lấy
0 = a ∈ K[p] ≤ H[p],doα là đẳng cấu nên tồn tại x ∈ G[p]:α(x)=a.
Khi đó, ta có a = α(x)=f(x) ∈ f(G) ∩ K =0(mâu thuẫn).
Vậy f(G)=H
Cách 2: Ta có
r
p
(G)=dim
Z
p
G[p]=dim

Z
p
H[p]=r
p
(H)
Đặt k = r
p
(G)=r
p
(H). Khi đó, ta có:
G =

i∈I
G
i
H =

i∈I
H
i
Trong đó G
i
,H
i
là các nhóm tựa cyclic kiểu p và đương nhiên chúng đẳng cấu
với nhau và |I| = k. Do đó, ta phải có G

=
H
Chú ý. Các nhóm cyclic không tầm thường đều không chia được.

Thật vậy, giả sử x là nhóm chia được, với x =0.
Đặng Tuấn Hiệp - Bài tập lý thuyết nhóm 18
1. Nếu |x| vô hạn, lấy n>0 bất kỳ, do x chia được nên tồn tại x

∈x sao cho
x = nx

= nmx với x

= mx. Suy ra (nm − 1)x =0⇒ nm − 1=0(vô lý)
2. Nếu |x| hữu hạn, lấy n = |x| thì khi đó x sẽ không chia được cho n bởi vì nếu
x = nx

thì x = nx

=0(vô lý)
Bài 4.1.8. Chứng minh rằng G chia được khi và chỉ khi G không có nhóm con tối đại.
Giải.
1. Giả sử G chia được. Ta sẽ chứng minh G không có nhóm con tối đại.
Giả sử ngược lại, G có nhóm con tối đại là M. Lấy x ∈ G\M. Do tính tối đại
của M, ta phải có M + x = G. Khi đó, G/M = x + M chia được, khác 0
(do x/∈ M). Suy ra x + M chia được (mâu thuẫn với chú ý trên).
2. Giả sử G không có nhóm con tối đại. Ta sẽ chứng minh G là nhóm chia được.
Muốn vậy, ta phải kiểm tra pG = G với mọi số nguyên tố p. Giả sử ngược lại,
tồn tại số nguyên tố p sao cho pG = G. Khi đó, ta có
p(G/pG)=
¯
0
Suy ra G/pG là không gian vectơ trên Z
p

.
Gọi {α
i
}
i∈I
là cơ sở của G/pG trên Z
p
.
Tương tự như bài 4.1.2, ta kiểm tra G = α
i
,pG
i∈I
.
Hiển nhiên, α
i
,pG
i∈I
⊂ G.
Lấy g ∈ G ⇒ ¯g ∈ G/pG, tồn tại m
1
, ,m
k
sao cho
¯g =
k

i=1
m
i
α

i
=
k

i=1
m
i
α
i
Suy ra
g −
k

i=1
m
i
α
i
∈ pG
g −
k

i=1
m
i
α
i
= pg
1
(g

1
∈ pG)
g =
k

i=1
m
i
α
i
+ pg
1
∈α
i
,pG
i∈I
Đặng Tuấn Hiệp - Bài tập lý thuyết nhóm 19
Ta sẽ chỉ ra trong G có một nhóm con tối đại.
Lấy i
0
∈ I cố đònh. Đặt J = I\{i
0
}, ta sẽ chứng minh H = α
i
,pG
i∈J

nhóm con tối đại của G.
Ta có α
i

0
/∈ H, vì nếu α
i
0
∈ H thì
α
i
0
=

i∈J
m
i
α
i
+ pg (g ∈ G)
Suy ra
α
i
0
=

i∈J
m
i
α
i
Điều này mâu thuẫn với tính độc lập của cơ sở {α
i
}

i∈I
.
Do đó H = G.
Lấy α/∈ H bất kỳ, ta sẽ chứng minh H, α = G.
Ta có α ∈ G = α
i
,pG
i∈I
⇒ α =

i∈I
m
i
α
i
+ pg, g ∈ G.
Do α/∈ H nên m
i
0
α
i
0
=0⇒ m
i
0
α
i
0
=
¯

0 ⇒ (m
i
0
,p)=1.
Không mất tính tổng quát, ta xét các m
i
α
i
trong tổng xác đònh α đều khác 0
(nếu có m
i
α
i
=0thì ta loại bỏ nó đi). Khi đó m
i
,p)=1. Ta có
m
i
0
α
i
0
= α −

i∈J
m
i
α
i
+ pg ∈H, α

Do (m
i
0
,p)=1nên tồn tại u, v ∈ Z :1=m
i
0
u + pv
Suy ra
α
i
0
= u(m
i
0
α
i
0
)+pvα
i
0
∈H, α
Do đó
H, α = G
Bài 4.1.9. Chứng minh rằng G chia được khi và chỉ khi G không có ảnh đồng cấu hữu
hạn khác 0.
Giải.
1. Giả sử G chia được. Lấy H là nhóm con thật sự của G. Ta cần chứng minh
G/H vô hạn. Thật vậy, nếu G/H hữu hạn thì G/H có hạng tử trực tiếp là
nhóm cyclic và hạng tử trực tiếp của G/H−chia được nên cũng chia được.
Điều này có nghóa là có tồn tại nhóm cyclic chia được (vô lý).

Đặng Tuấn Hiệp - Bài tập lý thuyết nhóm 20
2. Giả sử G không có ảnh đồng cấu hữu hạn khác 0. Ta cần chứng minh G chia
được. Muốn vậy, theo bài 4.1.8, ta sẽ chứng minh G không có nhóm con tối
đại. Giả sử phản chứng, G có nhóm con tối đại, suy ra G/M =0. Hơn nữa,
G/M chỉ có các nhóm con tầm thường là 0 và G/M. Thật vậy, vì nếu có H
là nhóm con thật sự của G/M thì 0 <H<G/M ⇒ M<π
−1
(H) <G, trong
đó π : G → G/M; x → x + M là phép chiếu tự nhiên (mâu thuẫn với tính
tối đại của M). Suy ra G/M là nhóm cyclic cấp nguyên tố nên G/M hữu hạn,
trái giả thiết. Vậy G không có nhóm con tối đại, suy ra G chia được.
Chú ý. Ta có G là nhóm cyclic cấp nguyên tố khi và chỉ khi G chỉ có hai nhóm
con là các nhóm con tầm thường.
Bài 4.1.10. Lấy ví dụ về hai nhóm con chia được nhưng có giao không chia được.
Giải. Trong nhóm nhân các số phức khác 0, xét các nhóm con:
• A = {2
r
|r ∈ Q}≤C

A chia được vì với mọi số nguyên dương n, ta có (2
r/n
)
n
=2
r
và 2
r/n
∈ A.
• B = {2
r

(cos 2πr + i sin 2πr) | r ∈ Q}≤C

B chia được vì với mọi α =2
r
(cos 2πr + i sin 2πr) ∈ B, với mọi số nguyên
dương n, ta chọn β =2
r/n
(cos
2πr
n
+ i sin
2πr
n
) ∈ B, khi đó ta sẽ có β
n
= α.
• Xét A ∩ B. Lấy x ∈ A ∩ B,x =2
r
=2
r
(cos 2πr + i sin 2πr) suy ra ta phải có
cos 2πr =1và sin 2πr =0, từ đó ta có r ∈ Z. Do đó
A ∩ B = {2
r
| r ∈ Z}

=
Z
nên không chia được.
4.2 Tổng trực tiếp các nhóm cyclic và tựa cyclic

Đề bài
1. Cho G là nhóm Abel tự do trên tập có n phần tử. Chứng minh rằng G không
thể được sinh bởi ít hơn n phần tử.
2. Cho G là nhóm Abel. Chứng minh rằng r(G) hữu hạn ⇔ max{d(H)} hữu
hạn, với H chạy khắp tập các nhóm con hữu hạn sinh của G, d(H) là số phần
tử của tập sinh tối tiểu của H. Chứng minh rằng trong trường hợp này ta có
r(G) = max{d(H)}.
Đặng Tuấn Hiệp - Bài tập lý thuyết nhóm 21
3. Cho G là nhóm Abel hữu hạn sinh. Chứng minh rằng d(G)=r(G). Hơn nữa,
d(G)=r
0
(G) ⇔ G không xoắn.
4. Cho A và B là các nhóm Abel hữu hạn sinh với B không xoắn. Chứng minh
rằng d(A ⊕ B)=d(A)+d(B).
5. Chứng minh rằng r(G) ≤ 1 ⇔ G đẳng cấu với nhóm con của Q hoặc Q/Z.
6. Cho G là nhóm Abel không xoắn. Chứng minh rằng r(G) ≤ r ⇔ G đẳng cấu
với một nhóm con của không gian vectơ r chiều trên Q.
7. Cho G là nhóm Abel và H ≤ G. Chứng minh rằng:
i. r
0
(H)+r
0
(G/H)=r
0
(G)
ii. r
p
(H)+r
p
(G/H) ≥ r

p
(G). Dấu ”=”xảy ra khi nào?
8. Cho A, B là các nhóm Abel và µ : A → B là đồng cấu, G là nhóm Abel không
xoắn. Chứng minh rằng µ

: A ⊗
Z
G → B ⊗
Z
G; a ⊗ g → µ(a) ⊗ g là một đồng
cấu.
9. Chứng minh rằng nhóm Abel G là tự do nếu và chỉ nếu G có tính chất: nếu
K ≤ H−Abel và H/K

=
G thì K là hạng tử trực tiếp của H.
10. Cho G là nhóm Abel hữu hạn sinh. Chứng minh rằng mọi toàn cấu  : G → G
đều là tự đẳng cấu.
11. Cho G là nhóm Abel thỏa điều kiện min. Chứng minh rằng mọi tự đơn cấu
của G đều là tự đẳng cấu.
12. Có bao nhiêu kiểu đẳng cấu của nhóm Abel cấp p
n
?
(Có bao nhiêu kiểu nhóm Abel cấp p
n
khác nhau?)
13. Cho G là nhóm Abel hữu hạn và m||G|. Chứng minh rằng G có một nhóm
con và một nhóm thương cấp m.
14. Cho G và H là hai nhóm Abel hữu hạn. Chứng minh rằng nếu với mọi số
m>0, G và H có cùng số phần tử cấp m thì G


=
H.
15. Chứng minh rằng nhóm Abel hữu hạn sinh có thặng dư hữu hạn.
Bài giải chi tiết
Bài 4.2.1. Cho G là nhóm Abel tự do trên tập có n phần tử. Chứng minh rằng G
không thể được sinh bởi ít hơn n phần tử.
Đặng Tuấn Hiệp - Bài tập lý thuyết nhóm 22
Giải. Ta có
G =
n

i=1
x
i

trong đó x
i
không xoắn ⇔x
i
 vô hạn.
•{x
1
,x
2
, ,x
n
} độc lập ⇔ nếu

n

i=1
m
i
x
i
=0thì m
i
x
i
=0⇒ m
i
=0(do x
i
không xoắn).
•{x
1
,x
2
, ,x
n
} là hệ sinh ⇔∀α ∈ G : α =

n
i=1
m
i
x
i
.
Gọi {x

1
,x
2
, ,x
n
} là hệ độc lập tối đại các phần tử không xoắn. Khi đó
n = r
0
(G)=r(G)(vì r
p
(G)=0, ∀p là số nguyên tố)
Số n cũng chính là số vectơ của cơ sở của không gian vectơ Q ⊗
Z
G trên Q.
Q × Q ⊗
Z
G −→ Q ⊗
Z
G
(r, s⊗ g) −→ (rs) ⊗ g
Cơ sở của Q ⊗
Z
G trên Q là hệ {1 ⊗ x
1
, 1 ⊗ x
2
, ,1 ⊗ x
n
}.
Do G không xoắn nên r(G)=r

0
(G)=dim
Q
Q ⊗
Z
G.
Nếu {α
1

2
, ,α
k
} là hệ sinh của G thì {1 ⊗ α
1
, 1 ⊗ α
2
, ,1 ⊗ α
k
} là hệ sinh của
Q ⊗
Z
G trên Q. Thật vậy:
k
l
⊗ g =
k
l
(1 ⊗ g)=
k
l

(1 ⊗
k

i=1
m
i
α
i
)=
k
l
k

i=1
(1 ⊗ m
i
α
i
)
=
k
l
k

i=1
m
i
(1 ⊗ α
i
)=

k

i=1
km
i
l
(1 ⊗ α
i
)
Trong không gian vectơ Q ⊗
Z
G, một hệ sinh luôn có ít nhất n = dim Q ⊗
Z
G phần
tử. Do đó, ta phải có k ≥ n
Chú ý. Kỹ thuật cơ bản để giải quyết các bài toán liên quan đến hạng của nhóm
Abel là ta thường chuyển từ nhóm Abel tự do G sang không gian Q ⊗
Z
G trên Q.
Cách khác. Sử dụng tính chất: Nếu B
i
≤ A
i
, với mọi i ∈ I thì ta luôn có:
(

i∈I
A
i
)/(


i∈I
B
i
)

=

i∈I
(A
i
/B
i
)
Đặng Tuấn Hiệp - Bài tập lý thuyết nhóm 23
Thật vậy, xét đồng cấu
π :

i∈I
A
i
−→

i∈I
(A
i
/B
i
)


i∈I
a
i
−→

i∈I
a
i
Khi đó, π là toàn cấu và kerπ =

i∈I
B
i
. Áp dụng đònh lý Noetherian, ta sẽ có
đẳng cấu
(

i∈I
A
i
)/(

i∈I
B
i
)

=

i∈I

(A
i
/B
i
)
Trở lại bài toán, ta có
G = x
1
⊕x
2
⊕···⊕x
n

Suy ra
pG = px
1
⊕px
2
⊕···⊕px
n

Do đó, áp dụng tính chất trên, ta sẽ có
G/pG

=
x
1
/px
1
⊕x

2
/px
2
⊕···⊕x
n
/px
n

Trong đó mỗi hạng tử trực tiếp x
i
/px
i
 đều có cấp là p.
Với mọi số nguyên tố p, ta luôn có G/pG là không gian vectơ trên Z
p
.
Hơn nữa, ta còn có
n = dim
Z
p
G/pG = r(G)=r
0
(G) = dim
Z
p
Q ⊗
Z
G
Nếu {α
1


2
, ,α
k
} là hệ sinh của G thì {α
1
+ pG, α
2
+ pG, ,α
k
+ pG} là hệ sinh
của không gian vectơ G/pG trên Z
p
. Thật vậy, lấy bất kỳ α + pG ∈ G/pG, ta có
α ∈ G và khi đó
α =
k

i=1
m
i
α
i
⇒ α + pG =
k

i=1
m
i
α

i
+ pG =
k

i=1
m
i

i
+ pG)
Do đó, ta phải có k ≥ n.
Bài 4.2.2. Cho G là nhóm Abel. Chứng minh rằng r(G) hữu hạn ⇔ max{d(H)} hữu
hạn, với H chạy khắp tập các nhóm con hữu hạn sinh của G. Chứng minh rằng trong
trường hợp này ta có r(G) = max{d(H)}.
Giải. Chiều thuận: Giả sử r(G)=n<∞.
Lấy H là nhóm con hữu hạn sinh của G. Theo bài 4.1.3, ta có
d(H)=r(H)=r
0
(H)+max
p
r
p
(H) ≤ r
0
(G) + max
p
r
p
(G)=r(G)=n<∞
Đặng Tuấn Hiệp - Bài tập lý thuyết nhóm 24

⇒ max d(H) ≤ n<∞
Trong trường hợp này, ta sẽ chứng minh max d(H)=n = r(G). Thật vậy, do
r(G)=n nên r
0
(G) + max
p
r
p
(G)=n. Đặt r
0
(G)=k ≤ n.
Lấy {α
1
, ,α
k
} là hệ độc lập tối đại gồm các phần tử không xoắn của G, {β
1
, ,β
l
}
là hệ độc lập tối đại các phần tử cấp lũy thừa của p, với l = max
p
r
p
(G),n= k + l.
Ta sẽ chứng minh {α
1
, ,α
k


1
, ,β
l
} là hệ độc lập của G. Giả sử ta có
m
1
α
1
+ ···+ m
k
α
k
+ n
1
β
1
+ ···+ n
l
β
l
=0
Gọi
p
k
= max
1≤i≤l

i
|
Ta có

k

i=1
p
k
m
i
α
i
=0⇒ p
k
m
i
α
i
=0⇒ p
k
m
i
=0⇒ m
i
=0, 1 ≤ i ≤ k
Suy ra
l

i=1
n
i
β
i

=0⇒ n
i
β
i
=0, 1 ≤ i ≤ l
Do đó {α
1
, ,α
k

1
, ,β
l
} là hệ độc lập của G.
Lấy
H = α
1
, ,α
k

1
, ,β
l
 = α
1
⊕···⊕α
k
⊕β
1
⊕···⊕β

l

Ta có
d(H)=r(H)=k + l = n
Vậy
max d(H)=n = r(G)
Chiều đảo: Giả sử ngược lại, n = max d(H) < ∞, ta sẽ chứng minh r(G) < ∞. Giả
sử phản chứng, r(G)=∞⇔r
0
(G)+maxr
p
(G)=∞. Xét các trường hợp:
• r
0
(G)=∞, suy ra tồn tại hệ {α
1
, ,α
n

n+1
} độc lập gồm các phần tử không
xoắn của G. Lấy H = α
1
, ,α
n

n+1
 = α
1
⊕···⊕α

n+1
. Khi đó
d(H)=r(H)=n +1>n= max d(H)
Ta có điều mâu thuẫn.

×